Find the solution set of the inequality and the number: 12 − 6x > 24 A. , C. ≤, D. ≥, E. =

Answers

Answer 1

Answer:

x < -2

Step-by-step explanation:

12 − 6x > 24

Subtract 12 from each side

12-12 − 6x > 24-12

-6x > 12

Divide each side by -6, remembering to flip the inequality

-6x/-6 < 12/-6

x < -2

Answer 2

Answer:

x < -2

Step-by-step explanation:

12 − 6x > 24

12 - 12 − 6x > 24 - 12

-6x > 12

-6x/(-6) < 12/(-6)

x < -2


Related Questions

if P(x)=1+6x-5x^2 represents the profit in selling x thousand Boombotix speakers, how many speakers should be sold to maximize profit?

Answers

Answer:

600

Step-by-step explanation:

[tex]p(x) = 1 + 6x - 5x^2[/tex]

x max = [tex]-b/2a[/tex]

a = -5

b = 6

-6/2(-5) = 6/10 = 3/5 = .6

.6 thousand = 600

600 speakers should be sold.

Alternatively, you can check the vertex of the parabola formed.

Find a polar equation r for the conic with its focus at the pole and the given eccentricity and directrix. (For convenience, the equation for the directrix is given in rectangular form.)
Conic: Parabola Eccentricity: e = 1 Directrix: y = 4

Answers

Answer:

The  equation is  [tex]r = \frac{4 }{ 1 + cos (\theta )}[/tex]

Step-by-step explanation:

From the equation we are told that

    The  Eccentricity: e = 1

    The  Directrix is   y = 4

Generally the polar equation for e =  1  and  y  =  + c is mathematically represented as

         [tex]r = \frac{e * c }{ 1 + ecos (\theta )}[/tex]

So

         [tex]r = \frac{1 * 4 }{ 1 + 1 * cos (\theta )}[/tex]

        [tex]r = \frac{4 }{ 1 + cos (\theta )}[/tex]

Solve for x: 3(x + 1)= -2(x - 1) + 6.

Answers

Answer:

x=1

Step-by-step explanation:

3(x + 1)= -2(x - 1) + 6.

Distribute

3x+3 = -2x+2+6

Combine like terms

3x+3 = -2x+8

Add 2x to each side

3x+3+2x = 8

5x+3 = 8

Subtract 3 from each side

5x =5

Divide by 5

x =1

A Markov chain has 3 possible states: A, B, and C. Every hour, it makes a transition to a different state. From state A, transitions to states B and C are equally likely. From state B, transitions to states A and C are equally likely. From state C, it always makes a transition to state A.

(a) If the initial distribution for states A, B, and C is P0 = ( 1/3 , 1/3 , 1/3 ), find the distribution of X2

(b) Find the steady state distribution by solving πP = π.

Answers

Answer:

A) distribution of x2 = ( 0.4167 0.25 0.3333 )

B) steady state distribution = [tex]\pi a \frac{4}{9} , \pi b \frac{2}{9} , \pi c \frac{3}{9}[/tex]

Step-by-step explanation:

Hello attached is the detailed solution for problems A and B

A) distribution states for A ,B, C:

Po = ( 1/3, 1/3, 1/3 )  we have to find the distribution of x2 as attached below

after solving the distribution

x 2 = ( 0.4167, 0.25, 0.3333 )

B ) finding the steady state distribution solving

[tex]\pi p = \pi[/tex]

below is the detailed solution and answers

A researcher measures daily driving distance from college and weekly cost of gas for a group of commuting college students. What kind of correlation is likely to be obtained for these two variables?

Answers

Answer:

There is a positive correlation between these two variables.

Step-by-step explanation:

Positive correlation is an association amid two variables in which both variables change in the same direction.  

A positive correlation occurs when one variable declines as the other variable declines, or one variable escalates while the other escalates.

As the distance covered by the vehicle increases the amount of gas consumed also increases. Thus, the weekly cost of gas will also increase.

Thus, there is a positive correlation between these two variables.

Use the model to show to help find the sum 0.34 plus 0.49

Answers

Answer/Step-by-step explanation:

The idea to use in solving this problem using the model, is to express the number of shaded boxes in fraction form.

Thus, the blue red shaded boxes has 34 boxes shaded out of 100 boxes. This represents [tex] \frac{34}{100} [/tex]. This will give us 0.34.

The other shaded boxes represents [tex] \frac{49}{100} = 0.49 [/tex].

Using the model, we can solve 0.34 + 0.49.

Add both fractions together.

[tex] \frac{34}{100} + \frac{49}{100} = \frac{34+49}{100} [/tex]

[tex] \frac{83}{100} = 0.83 [/tex]

algebra and trigonometry difference

Answers

Answer:

Algebra deals with knowing the value of unknown variables and functional relationships, while trigonometry touches on triangles, sides and angles and the relationship between them.

Algebra is more on polynomial equations, x and y while trigonometry more on sine, cosine, tangent, and degrees.

Trigonometry is much more complicated than algebra but algebra has its uses in our daily lives, be it calculating distance from point to another or determining the volume of milk in a milk container.

Step-by-step explanation:

Answer:

Although both Algebra II and Trigonometry involve solving mathematical problems, Algebra II focuses on solving equations and inequalities while Trigonometry is the study of triangles and how sides are connected to angles.

hope this answer helps u

pls mark as brainliest .-.

Find the product of the roots of the equation
xl-5x - 36 = 0

Answers

Answer:

Step-by-step explanation:

Hello, I assume that you mean

[tex]x^2-5x-36[/tex]

The product is -36.

[tex]x_1 \text{ and } x_2 \text{ are the two roots, we can write}\\\\(x-x_1)(x-x_2)=x^2-(x_1+x_2)x+x_1\cdot x_2[/tex]

So in this example, it means that the sum is 5 and the product is -36.

Thank you

y - 4= -2(x + 3)

Complete the missing value in
the solution to the equation.

(-3, _ )

Answers

Answer:

4

Step-by-step explanation:

i distributed the -2 to what's in the parentheses. that equal 0. I then moved the 4 to the zero so that it becomes positive. I just assumed that you were ask for Y

Step-by-step explanation:

y-4=-2(x+3)....eq(1)

y- 4= -2x-6

y=-2x-2...eq(2)

subtituting equation 2 in equation 1

(-2x-2)-4=-2x-6

-2x-6=-2x-6

=0

Divide. Write the quotient in lowest terms. 3 3/4 ÷ 5/7

Answers

Rewrite 3 3/4 as an improper fraction

3 3/4 = 15/4

Now you have

15/5 / 5/7

When you divide fractions, change the division to multiplication and flip the second fraction over:

15/4 x 7/5

Now multiply the top numbers together and the bottom numbers together:

( 15 x 7) / (4 x 5) = 105/20

Write as a proper fraction:

105/20 = 5 1/4

Answer: 21/4 or 5 1/4

Explanation:
3 3/4 divide by 5/7
= 15/4 x 7/5
= 105/20
= 21/4 or 5 1/4

What is the value of the product (3 – 2i)(3 + 2i)?

Answers

Answer:

13

Step-by-step explanation:

(3 - 2i)(3 + 2i)

Expand

(9 + 6i - 6i - 4i^2)

Add

(9 - 4i^2)

Convert i^2

i^2 = ([tex]\sqrt{-1}[/tex])^2 = -1

(9 - 4(-1))

Add

(9 + 4)

= 13

Answer:

13.

Step-by-step explanation:

(3 - 2i)(3 + 2i)

= (3 * 3) + (-2i * 3) + (2i * 3) + (-2i * 2i)

= 9 - 6i + 6i - 4[tex]\sqrt{-1} ^{2}[/tex]

= 9 - 4(-1)

= 9 + 4

= 13

Hope this helps!

A researcher at the University of Washington medical school believes that energy drink consumption may increase heart rate. Suppose it is known that heart rate (in beats per minute) is normally distributed with an average of 70 bpm for adults. A random sample of 25 adults was selected and it was found that their average heartbeat was 73 bpm after energy drink consumption, with a standard deviation of 7 bpm. In order to test belief at the 10% significance level, determine P-value for the test.

Answers

Answer:

Step-by-step explanation:

Given that:

mean μ = 70

sample size = 25

sample mean = 73

standard deviation = 7

level of significance = 0.10

The null hypothesis and the alternative hypothesis can be computed as follows:

[tex]\mathtt{H_o : \mu = 70} \\ \\ \mathtt{H_1 : \mu > 70 }[/tex]

The z score for this statistics can be calculated by using the formula:

[tex]z = \dfrac{X- \mu}{\dfrac{\sigma}{\sqrt{n}}}[/tex]

[tex]z = \dfrac{73- 70}{\dfrac{7}{\sqrt{25}}}[/tex]

[tex]z = \dfrac{3}{\dfrac{7}{5}}[/tex]

[tex]z = \dfrac{3 \times 5}{{7}{}}[/tex]

z = 2.143

At level of significance of 0.10

degree of freedom = n -1

degree of freedom = 25 - 1

degree of freedom = 24

The p - value from the z score at   level of significance of 0.10 and degree of freedom of 24 is:

P - value = 1 - (Z < 2.143)

P - value =  1 - 0.9839

P - value =  0.0161

Decision Rule: since P value is lesser than the level of significance, we reject the null hypothesis.

Conclusion: We conclude that energy drink consumption increases heart rate.

1. The mean performance score on a physical fitness test for Division I student athletes is 947 with a population standard deviation of 205. Select a random sample of 64 of these students. Hint: we have a sample so use the standard error. What is the probability the mean of the sample is below 900

Answers

Answer:

0.033316

Step-by-step explanation:

We use the z score formula to solve for this question.

Since we are given the number of samples in the question, our z score formula is given as:

z = (x-μ)/ S.E

where x is the raw score

μ is the sample mean

S.E is the Standard error.

x is the raw score = 900

μ is the sample mean = Population mean = 947

Standard error =

This is calculated as Population standard deviation/ √No of samples

= 205/√64.

= 205/8

= 25.625

We proceed to calculate the z score

z = (x-μ)/ S.E

z = 900 - 947/25.625

= -1.83415

Using the z score table for normal distribution,

P(x≤ z) = P(z ≤ -1.83) = P(x ≤ 900)

P(x<900) = 0.033316

Therefore, the probability the mean of the sample is below 900 is 0.033316

About how many feet are in 3.6 kilometers? 1 m = 39.37 in

Answers

Answer:

11811 feet

Step-by-step explanation:

Hope it helps!

There are about 11,812 feet in 3.6 kilometers.

To convert kilometers to feet, we need to use the conversion factor:

1 kilometer = 3,280.84 feet.

Now, to find how many feet are in 3.6 kilometers,

we can multiply 3.6 by the conversion factor:

So, 3.6 kilometers x 3,280.84 feet/kilometer

= 11,811.504 feet.

Thus, Rounded to a whole number, there are about 11,812 feet in 3.6 kilometers.

Learn more about Unit Conversion here:

https://brainly.com/question/14573907

#SPJ6

If 2( a^2 +b^2 ) = ( a+b)^2 , then
a. a+b =0
b. a =b
c. 2a =b
d. ab =0

Answers

Answer:

the answer is a=b

Step-by-step explanation:

pls help:Find all the missing elements:

Answers

Answer:

B = 48.7° , C = 61.3° , b = 12

Step-by-step explanation:

In order to find B we must first angle C

To find angle C we use the sine rule

That's

[tex] \frac{ |a| }{ \sin(A) } = \frac{ |c| }{ \sin(C) } [/tex]

From the question

a = 15

A = 70°

c = 14

So we have

[tex] \frac{15}{ \sin(70) } = \frac{14}{ \sin(C) } [/tex]

[tex] \sin(C) = \frac{14 \sin(7 0 ) }{15} [/tex]

[tex]C = \sin^{ - 1} ( \frac{14 \sin(70) }{15} ) [/tex]

C = 61.288

C = 61.3° to the nearest tenth

Since we've found C we can use it to find B.

Angles in a triangle add up to 180°

To find B add A and C and subtract it from 180°

That's

A + B + C = 180

B = 180 - A - C

B = 180 - 70 - 61.3

B = 48.7° to the nearest tenth

To find b we can use the sine rule

That's

[tex] \frac{ |a| }{ \sin(A) } = \frac{ |a| }{ \sin(B) } [/tex]

[tex] \frac{15}{ \sin(70) } = \frac{ |b| }{ \sin(48.7) } [/tex]

[tex] |b| = \frac{15 \sin(48.7) }{ \sin(70) } [/tex]

b = 11.9921

b = 12.0 to the nearest tenth

Hope this helps you

Select the best with the least expensive corn per ounce The choices are in the image

Answers

Answer:

B

Step-by-step explanation:

Option A:

1.50÷18≈0.0833

3.00÷36≈0.0833

4.50÷54≈0.0833

Option B:

0.75÷15=0.05

Option C:

2.20÷15=0.146

A random sample of 11 students produced the following data, where x is the hours spent per month playing games, and y is the final exam score (out of a maximum of 50 points). The data are presented below in the table of values.
x y
14 46
15 49
16 37
17 42
18 37
19 31
20 25
21 23
22 20
23 15
24 12
What is the value of the intercept of the regression line, b, rounded to one decimal place?

Answers

Answer:

b = - 3.7

Step-by-step explanation:

here are the data values:

x   y          XY        X²

14 46         644       196

15 49         735       225

16 37          592      256

17 42          714        289

18 37          666      324

19 31           589      361

20 25         500      400

21 23          483       441

22 20         440       484

23 15          345       529

24 12         288       576

now we are required to find the summation (total) of all values of X, Y, XY and X².

∑X = 209

∑Y = 337

∑XY = 5996

∑X² = 4081

The formular for finding b is given as:

b = n∑XY - (X)(Y) / n∑X² - (∑X)²

= 11(5996) - (209)(337) / 11(4081) - (209)²

= 65956 - 70433 / 44891 - 43681

= -4477/ 1210

= -3.7

The question asked us to find the value of b but we can  go further to find the equation of the regression line:

a = ∑Y - b∑X / n

= 337 - (-3.7)(209)/  11

=1110.3/11

= 100.94

the equation is:

Y = 100.94 - 3.7X

I hope you find my solution useful!

=

Twelve apples cost $2.00. How much will 50 apples cost?

Answers

Answer:

$8.33

Step-by-step explanation:

[tex]Solve \:using \: proportion\\\\12\:apples = \$ 2\\50\:apples = \$ x\\Cross \: Multiply\\\\12x = 100\\\\\\\frac{12x}{12} = \frac{100}{12} \\\\x = \$ 8.333[/tex]

Answer:

About $8.33.

Step-by-step explanation:

Write a proportion. Make sure the values line up horizontally:

[tex]\frac{12\text{ apples}}{\$2} =\frac{50\text{ apples}}{\$x}[/tex]

Cross multiply:

[tex]100=12x\\x=25/3\approx\$8.33[/tex]

Allied Corporation is trying to sell its new machines to Ajax. Allied claims that the machine will pay for itself since the time it takes to produce the product using the new machine is significantly less than the production time using the old machine. To test the claim, independent random samples were taken from both machines. You are given the following results.
New Machine Old Machine
Sample Mean 25 23
Sample Variance 27 7.56
Sample Size 45 36
As the statistical advisor to Ajax, would you recommend purchasing Allied's machine? Explain.

Answers

Answer:

z(s) is in the acceptance region. We accept H₀  we did not find a significantly difference in the performance of the two machines therefore we suggest not to buy a new machine

Step-by-step explanation:

We must evaluate the differences of the means of the two machines, to do so, we will assume a CI  of 95%, and as the interest is to find out if the new machine has better performance ( machine has a bigger efficiency or the new machine produces more units per unit of time than the old one) the test will be a one tail-test (to the left).

New machine

Sample mean                  x₁ =    25

Sample variance               s₁  = 27

Sample size                       n₁  = 45

Old machine

Sample mean                    x₂ =  23  

Sample variance               s₂  = 7,56

Sample size                       n₂  = 36

Test Hypothesis:

Null hypothesis                         H₀             x₂  -  x₁  = d = 0

Alternative hypothesis             Hₐ            x₂  -  x₁  <  0

CI = 90 %  ⇒  α =  10 %     α = 0,1      z(c) = - 1,28

To calculate z(s)

z(s)  =  ( x₂  -  x₁ ) / √s₁² / n₁  +  s₂² / n₂

s₁  = 27     ⇒    s₁²  =  729

n₁  = 45    ⇒   s₁² / n₁    = 16,2

s₂  = 7,56   ⇒    s₂²  = 57,15

n₂  = 36     ⇒    s₂² / n₂  =  1,5876

√s₁² / n₁  +  s₂² / n₂  =  √ 16,2  + 1.5876    = 4,2175

z(s) = (23 - 25 )/4,2175

z(s)  =  - 0,4742

Comparing z(s) and  z(c)

|z(s)| < | z(c)|  

z(s) is in the acceptance region. We accept H₀  we did not find a significantly difference in the performance of the two machines therefore we suggest not to buy a new machine

The very hight dispersion of values s₁ = 27 is evidence of frecuent values quite far from the mean

How hot does it get in Death Valley? The following data are taken from a study conducted by the National Park System, of which Death Valley is a unit. The ground temperatures (°F) were taken from May to November in the vicinity of Furnace Creek. Compute the mean, median, and mode for these ground temperatures. (Enter your answers to one decimal place.) 147 153 170 172 185 181 182 185 181 170 181 167 153 145

Answers

Answer:

Mean: 169.4

Median: 171

Mode: 181

Step-by-step explanation:

I first sorted the numbers by value, least to greatest.

145 147 153 153 167 170 170 172 181 181 181 182 185 185

We can see that 181 occurs the most, 3 times, so it's the mode.

The median of this set will be the middle number(s).

When we take away 6 numbers from both sides we are left with 170 and 172, and the mean of these two numbers is 171. So the median is 171.

We can add all the numbers and divide by 14 to get the mean.

[tex]147+153+170+172+185+181+182+185+181+170+181+167+153+145=2372\\\\2372\div14\approx169.4[/tex]

Hope this helped!

The grade appeal process at a university requires that a jury be structured by selecting individuals randomly from a pool of students and faculty.​ (a) What is the probability of selecting a jury of all​ students? (b) What is the probability of selecting a jury of all​ faculty? (c) What is the probability of selecting a jury of students and ​faculty

Answers

Correct question is ;

The grade appeal process at a university requires that a jury be structured by selecting eight individuals randomly from a pool of nine students and eleven faculty.​ (a) What is the probability of selecting a jury of all​ students? (b) What is the probability of selecting a jury of all​ faculty? (c) What is the probability of selecting a jury of six students and two ​faculty?

Answer:

A) 7.144 × 10^(-5)

B) 0.00131

C) 0.0367

Step-by-step explanation:

We are given;

Number of students = 9

Number of faculty members = 11

A) Now, the number of ways we can select eight students from 9 =

C(9, 8) = 9!/(8! × 1!) = 9

Also, number of ways of selecting 8 individuals out of the total of 20 = C(20,8) = 20!/(8! × 12!) = 125970

Thus, probability of selecting a jury of all​ students = 9/125970 = 7.144 × 10^(-5)

B) P(selecting a jury of all faculty) = (number of ways to choose 8 faculty out of 11 faculty)/(Total number of ways to choose 8 individuals out of 20 individuals) = [C(11,8)]/[C(20,8)] = (11!/(8! × 3!))/125970 = 0.00131

C) P(selecting a jury of six students and two faculty) = ((number of ways to choose 6 students out of 9 students) × (number of ways to choose 2 faculty out of 11 faculty))/(Total number of ways to choose 8 individuals out of 20 individuals) = [(C(9,6) × C(11,2)]/125970

This gives;

(84 × 55)/125970 = 0.0367

x power 8 + x power 4 + 1
factorize

Answers

Answer:

[tex]1(x {}^{8} + x {}^{4} + 1)[/tex]

Step-by-step explanation:

[tex]x {}^{8} + {x}^{4} + 1 =1( x {}^{8} + x {}^{2} + 1)[/tex]

Hope this helps ;) ❤❤❤

Let me know if there is an error in my answer.

Answer: See below

Explanation:

x^8 + x^4 + 1 = 0
(x^8 + x^4) + 1 = 0
x^4(x^4 + 1) + 1 = 0
(x^4 + 1)(x^4 + 1) = 0

If A and B are independent events with P( A) = 0.35 and P( B) = 0.55, then P( A| B) is:_________.
a. .19
b. 1.57
c. .64
d. .91

Answers

Answer:

P( A| B)= 0.35. None of the options are correct

Step-by-step explanation:

Two events A and B are said to be independent if the occurrence of one of the events does not affect the other occurring. For example, the event of tossing two coins is an independent event since they occur simultaneously. Two events are therefore independent if the following are true.

P(A|B) = P(A)

P(B|A) = P(B)

P(A and B) = P(A)P(B)

If A and B are independent events with P( A) = 0.35 and P( B) = 0.55,

then P( A| B) is a probability of A occurring provided that B has occurred. This is known as conditional probability for an independent event.

From the condition above for independent events, P(A|B) = P(A) and since P(A) =  0.35, hence P(A|B) =0.35

Choose the inequality that represents the following graph.

Answers

If x is represented by the blue, A is the correct answer.

Answer:

option a

Step-by-step explanation:

give person above brainliest :)

(-1, 4) and (-2, 2).

Answers

Answer:

Slope : 2

slope-intercept: y = 2x + 6

Point-slope (as asked): y - 4 = 2 (times) (x + 1)

standered: 2x - y = -6

Step-by-step explanation:

Algebra Review

Write an algebraic expression for each verbal expression.

1. the sum of one-third of a number and 27

2. the product of a number squared and 4

3. Write a verbal expression for 5n^3 +9.

Answers

Answer:

Step-by-step explanation:

1. The sum of one-third of a number and 27

= [tex]\frac{1}{3}\times x +27\\= 1/3x +27[/tex]

2. The product of a number squared and 4

[tex]Let\:the\:unknown\: number\: be \:x\\\\x^2\times4\\\\= 4x^2[/tex]

3.Write a verbal expression for 5n^3 +9.

The sum of the product and of 5 and a cubed number and 9

What is the rise over run for the slope -11/9

Answers

Answer:  11 down and 9 right

Step-by-step explanation:

Slope IS rise over run where the top number of the fraction (numerator) determines the vertical distance --> positive is up, negative is down

and the bottom number of the fraction (denominator) determines the horizontal distance --> positive is right, negative is left.

Given slope = -11/9

the numerator is -11 so the "rise" is  DOWN 11 units

the denominator is 9 so the "run" is RIGHT 9 units

how many solutions are there to this non-linear systems/graph a. one solution,b.two solutions,c.no solutions

Answers

Photo of the graph?

Find the distance between points P(5, 1) and Q(3, 4) to the nearest tenth.
3.6
5
9.4
13

Answers

Answer:

≈ 3.6

Step-by-step explanation:

Calculate the distance d using the distance formula

d = [tex]\sqrt{(x_{2}-x_{1})^2+(y_{2}-y_{1})^2 }[/tex]

with (x₁, y₁ ) = (P(5, 1) and (x₂, y₂ ) = Q(3, 4)

d = [tex]\sqrt{(3-5)^2+(4-1)^2}[/tex]

   = [tex]\sqrt{(-2)^2+3^2}[/tex]

   = [tex]\sqrt{4+9}[/tex]

    = [tex]\sqrt{13}[/tex] ≈ 3.6 ( to the nearest tenth )

Answer:

3.6

Step-by-step explanation:

Look above bru

Other Questions
Please reply to please!!!!No.11-14Thank you in advance:) Identify the factor relevant in each investigation ? HELP ME ILL GIV ROBUX Identify the property shown by the equation. 14 6 = 6 14 A. Commutative Property B. Associative Property C. Identity Property D. Distributive Property PLEASE HELP ME Question #5: Make a sentence by adding the correct group of words. A beautiful sunset _______. A.was seen Friday night B.seen Friday night C.Friday night "A type of laser medium that produces a visible blue-green wavelength beam that is absorbed by red-brown pigmented tissue, such as the red hemoglobin in blood, and by the cells that lie under the retina, is a(n)" |-6+8| = simplify the expression What three things are required for a fire to start? What element is depicted by the following electron configuration: Which two events from the passage move the plot forward as part of the rising action? The wives regret not cooking for their husbands. The wives scheme how to banish their sister-in-law. The wives refuse to fill the water pitcher in the magical river. The wives send the sister-in-law to a magical river. The wives chant in order to learn to cook. What happened to the Sudetenland as a result of the Munich Agreement? A car moving east at 45 km/h turns and travels west at 30 km/h. What is themagnitude and direction of the change in velocity?mahalle 1.11 Solve for x 3x - 4 = 2x - 10 Open the source notes, and review the notes for all three sources:Should We Terraform Mars?MarsWhy Mars?Consider and record your thoughts about these concepts:one common idea among the textsone idea from the texts that differshow the ideas better help you understand the topic and answer the question Should humans travel to Mars? Practice Question 25 Which one of the following is an effective method of evaluating a cost center? Compare actual total costs with flexible budget data. Compare actual controllable costs with static budget data. Compare actual controllable costs with flexible budget data. Compare the actual profit generated with expected profit. What number is equivalent to 9 1/2? what is meant by oesophagus The _____ is important to the cultural context of Nectar in a Sieve. Select all that apply.Muslim traditionhistory of Britainpartition of IndiaIndian caste system 4NH3(g) 5O2(g)4NO(g) 6H2O(g) Using standard thermodynamic data at 298K, calculate the free energy change when 1.81 moles of NH3(g) react at standard conditions. How would you define a cloud today? as a non-factor networking server any remote virtualized computing infrastructure Who did Pound look to as the founder of truly American poetry, and why?